PT13.S2.Q09 - In a mature tourist market...

HoneyCabbage-1-1HoneyCabbage-1-1 Core Member
edited May 2022 in Logical Reasoning 49 karma

I am having a hard time understanding why 'D' is the correct answer choice. I initially chose 'A' as my answer, yet obviously, this was incorrect. I was hoping someone could break it down for me and explain why 'D' is correct and why 'A' is incorrect. Thank you in advance

Comments

  • taraodonnell.mtaraodonnell.m Core Member
    5 karma

    With this fill in the blank question you have to add the additional piece of information in order to make the conclusion work. the conclusion is that these hotels cant increase their profits. We know that there are two ways to increase profits, building or improving. building has been accounted for and you cant build, therefore if a hotel is going to increase profits here they would need to improve their quality. But since we are trying to prove that these hotels cant increase profits we have to say something along the lines that these hotels can not improve their quality any more which is what D says, since not one would be willing to pay for such improvements

    If you take A to say that they cant kick any guests out in order to make improvements you can think of it as an investment, loose the $50 of the guest staying the one night in order to make $200 after renovations. At some point you will increase your profit.

Sign In or Register to comment.